Oakley babysits on weekends. He charges a flat fee of $12, plus an additional $5 for each hour that he babysits. How much money would Oakely make if he babysits for 4 hours?

Answers

Answer 1

Answer:

32

Since theres a flat fee he automatically starts at 12 since he gets 5 every hour for 4 hours (5×4) ending up at 20. You add the total and the flat fee 20+12= 32

Answer 2

Answer:

$32

Step-by-step explanation:

First you must set up the equation.

A flat fee of $12 means it will be added and $5 each hour for 4 hours so 5 will be multiplied by 4.

$12 + $5 * 4

12 + 5 * 4 (or 12 + (5 * 4), whatever makes you remember it better)

Remember PEMDAS (Parentheses, Exponents, Multiplication, Divisiom, Addition, Subtraction)

Multiplication comes before Addition so you get 12 + 20

12 + 20 = 32

Remember to put units, so you would get $32.


Related Questions

Help me with this answer I don’t it

Answers

Answer:

f(-2) = g(-2) this is the answer

The diagram shows a right-angled triangle.
xo
26 cm
17 cm
Find the size of angle x.
Give your answer correct to 1 decimal place.

Answers

Answer:

Diagram? I don't see a diagram.

Where is the diagram?

Step-by-step explanation:

Let V be the set of all 3x3 matrices with Real number entries, with the usual definitions of scalar multiplication and vector addition. Consider whether V is a vector space over C. Mark all true statements (there may be more than one).

a. The scalar closure axiom is satisfied
b. The additive inverse axiom is not satisfied
c The additive inverse axiom is satisfied
d. The additive closure axiom is not satisfied
e. The scalar closure axiom is not satisfied
f. The additive closure axiom is satsified
g. V is not a vector space over C
h. V is a vector space over C
i. The zero axiom is satisfied
j. The zero axiom is not satisfied

Answers

Answer:

the Scalar Closure axiom is not satisfiedV is not a Vector Space of CThe Additive Closure axiom is satisfied.

Step-by-step explanation:

According to the Question,

Given That, Let V be the set of all 3x3 matrices with Real number entries, with the usual definitions of scalar multiplication and vector addition. Consider whether V is a vector space over C.For V is a vector space over C and V is Set of 3x3 Matrices with Real entries.

Then, For any u,w ∈ V ⇒ u+w ∈ V

And u∈V and z∈C ⇒ z u ∈ V

So, If we take any z= i ∈ C

and V be any 3x3 matrices with Real entrices.

then, z,v ∉ V  ∴z,v Has Complex entries

So, the Scalar Closure axiom is not satisfied

Hence, V is not a Vector Space of C

Any u,w ∈ W ⇒ u+w ∈ V

So, The Additive Closure axiom is satisfied.

andrea is planning a birthday party and wants to include a cheese board with the desserts.
she reads online that she should have 110g of cheese per person ,but the cheese is sold in blocks of 500g
How many blocks of cheese should she buy to ensure that each guest can have 110g of cheese?​

Answers

Step-by-step explanation:

how many people in the party please ?

If f(x) = x -2 and g(x) = 2x – 6, then g(4)/f(3) =​

Answers

Answer:

Step-by-step explanation:

(2×4-6)/(3-2)=2

Answer:

[tex]{ \tt{f(x) = x - 2}} \\ { \bf{f(3) = 3 - 2 = 1}} \\ \\ { \tt{g(x) = 2x - 6}} \\ { \bf{g(4) = 2(4) - 6 = 2}} \\ \\ { \boxed{ \tt{ \frac{g(4)}{f(3)} = \frac{2}{1} = 2}}}[/tex]

Determine whether the following event is mutually exclusive or not mutually exclusive.

Choosing a student who is a mathematics major or a business major from a nearby university to participate in a research study. (Assume that each student only has one major.)

Answers

Answer:

The event is mutually exclusive.

Step-by-step explanation:

Mutually exclusive events are events that cannot exist simultaneously.

Thus, events that are not mutually exclusive can exist simultaneously.

Since each student only has one major, a single student cannot be both a mathematics major and a business major.

So, the event is mutually exclusive.

Simplify:......................................................

Answers

Answer:

...

Step-by-step explanation:...

The cut off number is 2x-1

A person walks 1/6 mile in 1/18 hour.

The person's speed is _ miles per hour.

Answers

This Is What I Got!

Hope This Helps! :)

Have A Good Day!!

And If You Can I Wouldn't Mind A Brainliest! :))

Answer:

Divide 1/6 miles to 1/12hour since u wanna find our miles per hour

So it’ll be : 1/6 / 1/12

= 1/6 x 12/1

= 2 miles

HELP HELP HELP
Solve this

Answers

Answer:

What is the cos theta for, i would use sin to solve for theta and then we would get 41.25 degrees.

Step-by-step explanation:

A group of 3 boys is sharing 4 small pans of rice krispie treats. What is the total amount of rice krispie treats each boy will get?

Answers

Answer:

each boy will get 1.33333333333 of a pan

Step-by-step explanation:

Answer:

Each boy in that group would get about 1 and 1 third of a pan of treats or 1.33 repeating

Step-by-step explanation:

To find this you would take the numbers of pans (4) and divide it by the number of boys (3).

4 / 3 = 1.33333

What is the area of this figure?

Answers

Answer:

90km² only if it is parallelogram

Step-by-step explanation:

base = 9km

height=10km

area of parallelogram = b x h

=9km x 10km

=90km²

Answer:

A = 90km2

Step-by-step explanation:

Area of a rhombus is:

1. A = s x h (if given side and height)

2. A = 1/2 a x b (if given lengths of diagonals)

3. A = s^2 sin A (if given side and length)

Therefore from your problem, height and side is given thus, you'll use number 1

A = s x h

A = 9km x 10km = 90km2

Six consecutive numbers add up to a total of 69.what is the highest of these numbers?

Answers

Answer:

14

Step-by-step explanation:

9+10+11+12+13+14=69

The [tex]HIGHEST[/tex] of these numbers is [tex]14[/tex]

Let the [tex]CONSECUTIVE[/tex] numbers be : [tex]a, a+1, a +2,a+3,a+4,a+5[/tex]

Taking the sum :

[tex]a + a + 1 + a + 2 + a + 3 + a + 4 + a + 5 = 69\\6a + 15 = 69\\6a = 69 - 15\\6a = 54\\a = 54 / 6\\a = 9[/tex]

[tex]HIGHEST[/tex] value = [tex]a + 5 = 9 + 5 = 14[/tex]

Hence, [tex]HIGHEST[/tex] value = [tex]14[/tex]

Learn more : https://brainly.com/question/15974141

Which number's estimate written as a single digit times a power of 10 will have a negative exponent?

Answers

105 i hope this helps if not then i’m sorry

The probability distribution for a random variable x is given in the table X: -5,-3,-2,0,2,3 Probability: .17,.13,.33,.16,.11,.10 Find the probability that X <_-3

Answers

Answer:

0.3 probability that [tex]x \leq -3[/tex]

Step-by-step explanation:

The probability distribution is given in the table.

Probability that x <= -3

The values that are -3 or lower are -3 and -5. So

[tex]P(X \leq -3) = P(X = -3) + P(X = -5)[/tex]

From the table:

[tex]P(X = -3) = 0.13, P(X = -5) = 0.17[/tex]. So

[tex]P(X \leq -3) = P(X = -3) + P(X = -5) = 0.13 + 0.17 = 0.3[/tex]

0.3 probability that [tex]x \leq -3[/tex]

Answer:0.3

Step-by-step explanation:

Please please help me please I really need help please just tap on picture and you will see the question

Answers

Answer:

No , it is not a right angle triangle

Step-by-step explanation:

according to the pythagoras theorem in right angled triangle sum of square of two sides is equal to the square of it's hypotenuse.

using pythagoras theorem

a^2 + b^2 = c^2

9^2 + 16^2 = 25^2

81 + 256 = 625

337 = 625

since sum of square of two smallest sides of a triangle is not equal to the square of it's hypotenuse it can be concluded that the given figure does not form right angle triangle.

11x+7y=17
solve for y

Answers

[tex]\implies {\blue {\boxed {\boxed {\purple {\sf {\: y = \frac{17 - 11x}{7} }}}}}}[/tex]

[tex]\large\mathfrak{{\pmb{\underline{\red{Step-by-step\:explanation}}{\red{:}}}}}[/tex]

[tex]\\11x + 7y = 17[/tex]

[tex] \\➺ \: 7y = 17 - 11x[/tex]

[tex]\\➺ \: y = \frac{17 - 11x}{7} [/tex]

[tex]\bold{ \green{ \star{ \orange{Mystique35}}}}⋆[/tex]

he speeds (in MPH) of automobiles traveling in a city are given below:
20, 35, 42, 52, 65, 49, 24, 37, 23, 41, 50, 58
The mean speed of the cars is

Answers

Answer:

Mean speed = 41.3 mph

Step-by-step explanation:

Given that,

The speeds of an automobiles are given below:

20, 35, 42, 52, 65, 49, 24, 37, 23, 41, 50, 58

We need to find the mean speed of the cars.

Mean = sum of observations/ no. of observation

[tex]M=\dfrac{20+35+42+52+65+49+24+37+23+41+50+58}{12}\\\\M=41.3[/tex]

So, the mean speed of the cars is equal to 41.3 mph.

Joe drives for 3 hours and covers 201 miles. In miles per hour, how fast was he driving?​

Answers

Answer:

67 mph

Step-by-step explanation:

201/3 = 67

 Marsha has a bag that contains 4 green marbles, 8 yellow marbles , and 20 red marbles . If she chooses one marble from the bag, what is the probability that the marble is not red?

PLEASE HELP IF YOURE GOOD AT GEOMETRY!!

Answers

Answer:

C. 3/8

HOPE THIS HELPS :)

Answer:

c. 3/8

Step-by-step explanation:

first you need the denomerator by adding all marbles together which equals 32. now for the munerator you need the sum of the green and yellow marbles. this equals 12. so your fraction is 12/32. next we simplify. we can divide both numbers by 4. getting us a fraction of 3/8.

2 divided by 0.75 full divison work i dont just need the answer​

Answers

Answer:

0.375

Step-by-step explanation:

Check the picture below.

whenever we do division of decimals, we have to mind how many decimals are there on each amount, the dividend as well as the divisor, that way we pad with zeros the other amount accordingly whilst losing the dot, for example, to say divide 3 by 0.123, 3 has no decimals, whilst 0.123 has three decimals, so we can just divide 3000 by 0123, so dividing 3 by 0.123 is the same as dividing 3000 by 123.  Another example, if we were to divide say 23.761 by 555.89331, the dividend has 3 decimals, that means 3 zeros the other way, the divisor has 5 decimals, that means 5 zeros the other way while losing the dots, so we'd end up dividing 2376100000 by 55589331000, which we can simplify to just 2376100 by 5589331, as you can see in the picture in this case.

Heyy!! Can someone help me please!!

3 (5x + 2) - 2 (4x -4)

I don’t know what to doooo!!

Answers

Answer:

7x + 14

Step-by-step explanation:

the first thing to do is expand the parentheses/brackets.

3(5x + 2) -2(4x - 4) will be

3(5x) + 3(2) -2(4x) -2(-4)

= 15x + 6 - 8x + 8

collect like terms

15x - 8x + 6 + 8 = 7x + 14

the answer is 7x + 14

Answer:

3(5x+2)-2(4x-4)

15x+6-8x+8

15x-8x+6+8

7x+14

N/A questions does not exist

Answers

Answer:

ok

Step-by-step explanation:

why did you put it

Given points (-3;-6), G(3; -2) and H(6; 1); determine:
(a) The equation of line FG

Answers

Answer:

The equation of line FG is [tex]y = \frac{2}{3}x - 4[/tex]

Step-by-step explanation:

Equation of a line:

The equation of a line has the following format:

[tex]y = mx + b[/tex]

In which m is the slope and b is the y-intercept.

F(-3;-6), G(3; -2)

When we have two points, the slope is given by the change in y divided by the change in x. So

Change in y : -2 - (-6) = -2 + 6 = 4

Change in x: 3 - (-3) = 3 + 3 = 6

Slope: [tex]m = \frac{4}{6} = \frac{2}{3}[/tex]

So

[tex]y = \frac{2}{3}x + b[/tex]

Finding b:

(3; -2) means that when [tex]x = 3, y = -2[/tex]. We use this to find b.

[tex]y = \frac{2}{3}x + b[/tex]

[tex]-2 = \frac{2}{3}(3) + b[/tex]

[tex]2 + b = -2[/tex]

[tex]b = -4[/tex]

The equation of line FG is [tex]y = \frac{2}{3}x - 4[/tex]

Which diagram represents the hypothesis of the converse of corresponding angles theorem?

Answers

Answer:

the first diagram

Step-by-step explanation:

first one

international system of 89643092 in words​

Answers

Answer:

Eighty nine million six hundred forty three thousand ninety two

Step-by-step explanation:

89,643,092

=> Eighty nine million six hundred forty three thousand ninety two

Suppose that g(x)= f(x)+ 6. Which statement best compares the graph of g(x) with the graph of f(x)?

A. The graph of g(x) is the graph of f(x) shifted 6 units down.

B. The graph of g(x) is the graph of f(x) shifted to the right.

C. The graph of g(x) is the graph of f(x) shifted 6 units to the left.

D. The graph of g(x) is the graph of f(x) shifted 6 units up.

Answers

Answer:

D

Step-by-step explanation:

The + 6 moves it up 6 units.

The correct answer is (D) "The graph of g(x) is the graph of f(x) shifted 6 units up."

What is the function?

A relationship between a group of inputs and one output is referred to as a function. In plain English, a function is an association between inputs in which each input is connected to precisely one output. A domain, codomain, or range exists for every function. Typically, f(x), where x is the input, is used to represent a function.

When we add a constant to a function, such as in the case of g(x) = f(x) + 6, it will shift the graph of f(x) upward by 6 units.

This is because, for any value of x, the value of f(x) will be added to 6, resulting in a vertical shift of the entire graph.

Option (A) is incorrect because adding 6 to f(x) would shift the graph up, not down.

Option (B) is incorrect because adding a constant to a function does not cause it to shift horizontally.

Option (C) is incorrect because adding 6 to f(x) would shift the graph right, not left.

D. The graph of g(x) is the graph of f(x) shifted 6 units up. Adding a constant term to a function will shift the graph of the function vertically. In this case, adding 6 to f(x) will shift the graph of f(x) upward by 6 units, resulting in the graph of g(x).

Learn more about function here:

https://brainly.com/question/29633660

#SPJ7

Complete the table of ordered pairs for the linear y=2x-8

Answers

Given:

Consider the below figure attached with this question.

The linear equation is:

[tex]y=2x-8[/tex]

To find:

The values to complete the table of ordered pairs for the given linear equation.

Solution:

We have,

[tex]y=2x-8[/tex]

Substituting [tex]x=0[/tex] in the given equation, we get

[tex]y=2(0)-8[/tex]

[tex]y=0-8[/tex]

[tex]y=-8[/tex]

So, the value for first blank is -8.

Substituting [tex]y=-2[/tex] in the given equation, we get

[tex]-2=2x-8[/tex]

[tex]-2+8=2x[/tex]

[tex]\dfrac{6}{2}=x[/tex]

[tex]3=x[/tex]

So, the value for second blank is 3.

Substituting [tex]x=2[/tex] in the given equation, we get

[tex]y=2(2)-8[/tex]

[tex]y=4-8[/tex]

[tex]y=-4[/tex]

So, the value for third blank is -4.

Therefore, the required complete table is:

     x           y

     0         -8

    3         -2

     2          -4

Find the area of the following shape:

Answers

Answer:

36cm^2

Step-by-step explanation:

total area: 6x(4+3)=42

total area excluding the space: 42-(2x3)=36

Answer:

36 cm squared

Step-by-step explanation:

To solve this problem, I first construct a line. (shown in yellow in the first photo)

I then find the area of the top rectangle. (6 cm * 4 cm = 24 cm squared.)

Next, I find the area of the lower rectangle. But...to do that I have to find the length of the line that I constructed. To do this, I do  6cm-2cm=4cm.

Then I can find the area of the lower rectangle. (4cm*3cm=12cm squared.)

add up the area of both of the rectangles and.........12+24=36 cm squared

Which rules of exponents will be used to evallate this expression? Select three options.

Answers

I need the answer choices and the expression you’re referring to

Câu 1 (2 điểm). Cho hệ các vector

U = {(1,2,2); (0,-1,-1);(2,3,3);(1,0,0).

a) tìm số chiều và một cơ sở W của không gian con sinh bởi hệ vector U

b) tìm tham số k để u=(2,3,k^2 +1) là một tổ hợp tuyến tính cảu W, và suy ra [u]w

giải hộ mình với

Answers

Answer:

Step-by-step explanation:

Other Questions
RJM Enterprises is a manufacturer of consumer electronics products. The industry is very competitive, and RJM has seen its profits fall in recent years, including an operating loss of $16,328 last year. RJM was able to turn that around this year by aggressively cutting costs. The summarized financial results for RJM are shown below: How do people in your country protect thier jewelry? IELTS Speaking part 1 The Coffee Counter charges $8.00 per pound for Kenyan French Roast coffee and $10.00 per pound for Sumatran coffee. How much of each type should be used to make an 18 pound blend that sells for $9.00 per pound?The Coffee Counter should mix ___ pounds of Kenyan Roast coffee and ___ pounds of Sumatran coffee to make 18 pounds of a blend that sells for $9.00 per pound. the acceleration of a moving vehicles is 10 metre per second square what does it means In the triangles, GK PN and HG 2MP.Which statement correctly compares the angles?HMAngle G is congruent to angle P.O Angle G is smaller than angle P.O Angle G is larger than angle P.O Angle G is congruent to angle N.32 cm40 cmGKPN distance= 10km due West in 1hour calculate the velocity What is microorganisms Determine whether the following equation defines y as a function of x.x + y = 27Does the equation x + y = 27 define y as a function of x?O Yes $12.80 nearest cent these marbles are placed in a bag and two of them are randomly drawn. what is the probability of drawing two blue marbles if the first marble is placed back in the bag before the second draw Red Co. recorded a right-of-use asset of $140,000 in a 10-year finance lease. Payments of $22,784 are made annually at the end of each year. The interest rate charged by the lessor and known by Red was 10%. The balance in the lease payable after two years will be: (Round your final answer to the nearest whole dollar.) Which points are solutions to the system of inequalities shown below?Check all that apply.< xy< 5X advantanges of oral literature quien quiere ser mi novia Use the drop-down menu to complete thesentence.are capable of differentiation toform specialized cells.DONE Source GeoSystems Global Corporation (adapted)Whch group of people ruled much of Asia during the period shown on this map?1. Mongol 2. Indian3. Japanese 4. EuropeanSo a8TH GRADE HISTORY- MEXICAN AMERICAN WARDescribe events that explain the border differences between these maps. (USE THE MAP ABOVE TO EXPLAIN THE EVENTS). e. Use complete sentences.I really just need the Lonestar Republic but if you can do all I will give brainliest!Remember to write about it when it was in the time it says on the map Answer all the qustion get 10 points and get marked as a brainlist!!1.The Mississippi River and the St. Lawrence Seaway give the Midwest access to the ocean.TrueFalse2.British laws were unfair to the colonists.TrueFalse3.Colonists in the thirteen colonies refused to pay taxes to England.TrueFalse4.Mexico grew dissatisfied with Spanish rule and wanted:independenceEnglish ruleits own king5.After independence, the first Mexican rulers were:dictatorsmonarchspresidents6.Today, Mexico is governed by a:dictatorpresidentmonarch7.Mexican culture is a combination of and original Native American influences.FrenchEnglishSpanishPortuguese8.Immigration and urbanization were a result of:industrializationtransportationcommunication9.The United States acquired territory during:urbanizationexpansionindustrialization10.The first major territory obtained by the United States was:the Gadsden PurchaseTexasthe Louisiana Purchase Please help Ill give brainliest A circle has a circumference of 28 centimeters. If an arc subtends a central angle of 55 degrees, what is the arc length?A.4.28 centimetersB.26.88 centimetersC.53.76 centimetersD.8.56 centimeters